From: Dik T. Winter on
In article <689545.1172620604015.JavaMail.jakarta(a)nitrogen.mathforum.org> bassam king karzeddin <bassam(a)ahu.edu.jo> writes:

Please, do retain attributions of the articles you are responding to.

Randy Poe:
> > > All right, you didn't answer but somebody else e did. I have
> > > now seen a validation of this identity including g the
> > > statement that N(x,y,z) = 1 for p = 3.

> > See my proof of it in my previous article. For p = 1, N(x,y,z) = 0 and
> > for p = 3, N(x, y, z) = 1. That is also easily proven: (x + y + z)^p
> > - x^p - y^p - x^p is a homogenous polynomial of degree 3, and so it is
> > 3.(x + y)(x + z)(y + z). For p= 5 it is:
> > x^2 + x.y + x.z + y^2 + y.z + z^2.
>
> Where is your previous article please

In the newsgroup <news:JE3IDr.2A5(a)cwi.nl>.
--
dik t. winter, cwi, kruislaan 413, 1098 sj amsterdam, nederland, +31205924131
home: bovenover 215, 1025 jn amsterdam, nederland; http://www.cwi.nl/~dik/
From: Hisanobu Shinya on
> > > > Hello TO ALL
> > > >
> > > > The only remaining case -PROOF
> > > >
> > > > Now, you may easily derive the following
> > identity
> > > for
> > > > a counter example when (p) is a prime factor
> OF
> > > > (x*y*z)
> > > >
> > > > Let x^p + y^p = z^p, where (x,y,z) are three
> > > positive
> > > > coprime integers, then the following identity
> > must
> > > > hold
> > > >
> > > > (x*y*z)^p =
> > (x+y)*(z-x)*(z-y)*N(x,y,z)
> > >
> > > > where
> > > >
> > > >
> > > Gcd((x+y)*(z-x)*(z-y), N(x,y,z))= p,
> > >
> > > > Very Simple to prove, (no need to waste time
> on
> > > > that)
> > > >
> > > > Now, (p^k*p) divides the left hand side of the
> > > above
> > > > identity, where as p^(k*p-1) only divides the
> > > right
> > > > hand side of the above Identity, where k is
> > > positive
> > > > integer number,and as I had shown you today
> from
> > > > other post.
> > > >
> > > > Therefore, the identity can't hold true with
> > > integer
> > > > numbers defined above AND THERE IS NO COUNTER
> > > > EXAMPLE
> > > >
> > > > Hence THE PROOF IS INDEED COMPLETED
> > >
> > > SORRY, a flaw was found by me few hours after
> > posting
> > > proof of this case and it is no more valid, but
> I
> > > have another one
> >
> > I will be happy to read another argument.
> >
> > >
> > > > You may add any suitable references, and then
> > > write
> > > > it in any suitable languge, beside let the
> > > JOURNALS
> > > > KNOW ABOUT IT
> > > >
> > >
> > > >
> > >
> > > >
> > > > MY BEST REGARDS
> > > >
> > > > Bassam Karzeddin
> > > >
> > > > AL Hussein bin Talal University
> > > >
> > > > JORDAN
>
> I have already corrected that argument

Actually, I had not even asked what the mistake was.

>
> B.Karzeddin
From: Hisanobu Shinya on
> > > > > > > Fermat's Last theorem short proof
> > > > > > >
> > > > > > > We have the following general equation
> > > (using
> > > > > the
> > > > > > > general binomial theorem)
> > > > > > >
> > > > > > > (x+y+z)^p =p* (x+y)*(x+z)*(y+z)*N(x,y,z)
> +
> > > > > > > x^p+y^p+z^p
> > > > > > >
> > > > > > > Where
> > > > > > > N (x, y, z) is integer function in terms
> > of
> > > > (x,
> > > > > y,
> > > > > > z)
> > > > > > >
> > > > > > > P is odd prime number
> > > > > > > (x, y, z) are three (none zero) co prime
> > > > > integers?
> > > > > > >
> > > > > > > Assuming a counter example (x, y, z)
> > exists
> > > > such
> > > > > > that
> > > > > > > (x^p+y^p+z^p=0)
> > > > > > >
> > > > > > >
> > > > > > > (x+y+z)^p =p* (x+y)*(x+z)*(y+z)*N (x, y,
> z)
> >
> > > > > > >
> > > > > > > CASE-1
> > > > > > > If (p=3) implies N (x, y, z) = 1, so we
> > have
> > >
> > > > > >
> > > > > > Why?
> > > > >
> > > > > All right. I got this, and I think this is
> > > clever.
> > > > >
> > > > > >
> > > > > > >
> > > > > > > (x+y+z)^3 =3* (x+y)*(x+z)*(y+z)
> > > > > > >
> > > > > > > Assuming (3) does not divide (x*y*z),
> then
> > > it
> > > > > does
> > > > > > > not divide (x+y)*(x+z)*(y+z),
> > > > > >
> > > > > > Why?
> > > > > >
> > > > > > If x = 4, y = 5, z = 7, for instance, then
> > > > > >
> > > > > > x + y is divisible by 3, which is
> relatively
> > > > prime
> > > > > to
> > > > > > xyz.
> > > > >
> > > > > Besides, 3 is also a divisor of another
> factor
> > > (y
> > > > +
> > > > > z).
> > > >
> > > > My couterexample above is not sufficient to
> > > disprove
> > > > your argument, since it can be easily shown by
> > > > Fermat's Little Theorem and the counterexample
> > to
> > > > Fermat's Last Theorem that
> > > >
> > > > x + y = z mod 3;
> > > >
> > > > hence, if x + y is divisible by 3, then so is
> z,
> > > > contradictory to the first case assumption.
> > > >
> > > > However, I would like to present the following
> > > > example:
> > > >
> > > > x = 7, y = 10, z = 47.
> > > >
> > > > x + y = 17 is not divisible by 3;
> > > >
> > > > x + z = 54 = 3*18;
> > > >
> > > > y + z = 57 = 3*19.
> > > >
> > > > I hope I did the arithmetic right.
> > > >
> > >
> > > It is well established that a counter example
> MUST
> > > form a Triangle
> > >
> > > In my previous posts, I stated that a counter
> > example
> > > must form an integer triangle (with one side
> only
> > > even integer), and the largest angle between
> > (PI/2,
> > > PI/3)
> > >
> > > So, you see it follows directly
> > >
> > > Yours are not a counter example
> > >
> >
> >
> > Where is the previous post?
> >
> >
> > > My Regards
> > >
> > > Bassam Karzeddin
> > > AL Hussein bin Talal University
> > > JORDAN
>
> You may see at the following link, even I may have
> posted it befor
>
> http://mathforum.org/kb/message.jspa?messageID=4695333
> &tstart=0

So, should we focus on the validity of an argument in this old thread at this moment?


>
> My Regards
>
> Bassam Karzeddin
From: bassam king karzeddin on
Hello Dear

The theme is that I have realised that I have solved FLT, EVEN (16) YEARS BEFORE I KNEW ABOUT IT.

YOU may wonder how!!

IF S^P + M^P =L^P

My turn is a formula for P
Your turn is to know why it doesn't work for integers > 2

so, simple question after a formula is being found, but it was not suitable for them MANY YEARS BACK

Now, I realised that somthing WRONG happeining world wise in the holly science-MATHEMATICS, AND, PYTHAGOURS, FERMAT, ...,ARE ALL very , very, VERY ANGRY

THEN, WITH YOUR HELP, we are going to CHANGE THE RULES

My Regards

Bassam Karzeddin
Al Hussein bin Talal University
JORDAN
From: bassam king karzeddin on
> > > > > Hello TO ALL
> > > > >
> > > > > The only remaining case -PROOF
> > > > >
> > > > > Now, you may easily derive the following
> > > identity
> > > > for
> > > > > a counter example when (p) is a prime factor
> > OF
> > > > > (x*y*z)
> > > > >
> > > > > Let x^p + y^p = z^p, where (x,y,z) are three
> > > > positive
> > > > > coprime integers, then the following
> identity
> > > must
> > > > > hold
> > > > >
> > > > > (x*y*z)^p =
> > > (x+y)*(z-x)*(z-y)*N(x,y,z)
> > > >
> > > > > where
> > > > >
> > > > >
> > > > Gcd((x+y)*(z-x)*(z-y), N(x,y,z))= p,
> > > >
> > > > > Very Simple to prove, (no need to waste time
> > on
> > > > > that)
> > > > >
> > > > > Now, (p^k*p) divides the left hand side of
> the
> > > > above
> > > > > identity, where as p^(k*p-1) only divides
> the
> > > > right
> > > > > hand side of the above Identity, where k is
> > > > positive
> > > > > integer number,and as I had shown you today
> > from
> > > > > other post.
> > > > >
> > > > > Therefore, the identity can't hold true with
> > > > integer
> > > > > numbers defined above AND THERE IS NO
> COUNTER
> > > > > EXAMPLE
> > > > >
> > > > > Hence THE PROOF IS INDEED COMPLETED
> > > >
> > > > SORRY, a flaw was found by me few hours after
> > > posting
> > > > proof of this case and it is no more valid,
> but
> > I
> > > > have another one
> > >
> > > I will be happy to read another argument.
> > >
> > > >
> > > > > You may add any suitable references, and
> then
> > > > write
> > > > > it in any suitable languge, beside let the
> > > > JOURNALS
> > > > > KNOW ABOUT IT
> > > > >
> > > >
> > > > >
> > > >
> > > > >
> > > > > MY BEST REGARDS
> > > > >
> > > > > Bassam Karzeddin
> > > > >
> > > > > AL Hussein bin Talal University
> > > > >
> > > > > JORDAN
> >
> > I have already corrected that argument
>
> Actually, I had not even asked what the mistake was.

Dear Hisanobu Shinya

It was not leading to the result I desire, so, let it go...

My Regards
>
> >
> > B.Karzeddin